Difference between revisions of "2008 iTest Problems/Problem 39"

(Solution to Problem 39)
 
m (Solution)
Line 9: Line 9:
  
 
Using the formula <math>\phi(n) = n \cdot (1 - \tfrac{1}{p_1})(1 - \tfrac{1}{p_2}) \cdots (1 - \tfrac{1}{p_n})</math> (or by manually counting the numbers relatively prime to each number), <math>1</math> number is relatively prime to <math>1</math>, <math>1</math> number is relatively prime to <math>2</math>, <math>2</math> numbers are relatively prime to <math>4</math>, <math>4</math> numbers are relatively prime to <math>8</math>, <math>250</math> numbers are relatively prime to <math>251</math>, <math>250</math> numbers are relatively prime to <math>502</math>, <math>500</math> numbers are relatively prime to <math>1004</math>, and <math>1000</math> numbers are relatively prime to <math>2008</math>.  That means <math>S = 2008</math>, and the remainder when <math>S</math> is divided by <math>1000</math> is <math>\boxed{8}</math>.
 
Using the formula <math>\phi(n) = n \cdot (1 - \tfrac{1}{p_1})(1 - \tfrac{1}{p_2}) \cdots (1 - \tfrac{1}{p_n})</math> (or by manually counting the numbers relatively prime to each number), <math>1</math> number is relatively prime to <math>1</math>, <math>1</math> number is relatively prime to <math>2</math>, <math>2</math> numbers are relatively prime to <math>4</math>, <math>4</math> numbers are relatively prime to <math>8</math>, <math>250</math> numbers are relatively prime to <math>251</math>, <math>250</math> numbers are relatively prime to <math>502</math>, <math>500</math> numbers are relatively prime to <math>1004</math>, and <math>1000</math> numbers are relatively prime to <math>2008</math>.  That means <math>S = 2008</math>, and the remainder when <math>S</math> is divided by <math>1000</math> is <math>\boxed{8}</math>.
 +
 +
==Extra Note==
 +
If the divisors of n are <math>{d_1}</math>,  <math>{d_2}</math>,  <math>{d_3}</math>, ... <math>{d_k}</math>,  <math>\phi(n)</math>  =  <math>\phi({d_1})</math> +  <math>\phi({d_2})</math> +  .... +  <math>\phi({d_k})</math>.
 +
This provides a shorter approach in this problem because it asks about
  
 
==See Also==
 
==See Also==

Revision as of 15:43, 20 August 2023

Problem

Let $\phi(n)$ denote $\textit{Euler's phi function}$, the number of integers $1\leq i\leq n$ that are relatively prime to $n$. (For example, $\phi(6)=2$ and $\phi(10)=4$.) Let $S=\sum_{d|2008}\phi(d)$, in which $d$ ranges through all positive divisors of $2008$, including $1$ and $2008$. Find the remainder when $S$ is divided by $1000$.

Solution

The divisors of $2008$ are $1$,$2$,$4$,$8$,$251$,$502$,$1004$, and $2008$, so the problem requires the sum of the number of numbers relatively prime to each of the eight numbers.

Using the formula $\phi(n) = n \cdot (1 - \tfrac{1}{p_1})(1 - \tfrac{1}{p_2}) \cdots (1 - \tfrac{1}{p_n})$ (or by manually counting the numbers relatively prime to each number), $1$ number is relatively prime to $1$, $1$ number is relatively prime to $2$, $2$ numbers are relatively prime to $4$, $4$ numbers are relatively prime to $8$, $250$ numbers are relatively prime to $251$, $250$ numbers are relatively prime to $502$, $500$ numbers are relatively prime to $1004$, and $1000$ numbers are relatively prime to $2008$. That means $S = 2008$, and the remainder when $S$ is divided by $1000$ is $\boxed{8}$.

Extra Note

If the divisors of n are ${d_1}$, ${d_2}$, ${d_3}$, ... ${d_k}$, $\phi(n)$ = $\phi({d_1})$ + $\phi({d_2})$ + .... + $\phi({d_k})$. This provides a shorter approach in this problem because it asks about

See Also

2008 iTest (Problems)
Preceded by:
Problem 38
Followed by:
Problem 40
1 2 3 4 5 6 7 8 9 10 11 12 13 14 15 16 17 18 19 20 21 22 23 24 25 26 27 28 29 30 31 32 33 34 35 36 37 38 39 40 41 42 43 44 45 46 47 48 49 50 51 52 53 54 55 56 57 58 59 60 61 62 63 64 65 66 67 68 69 70 71 72 73 74 75 76 77 78 79 80 81 82 83 84 85 86 87 88 89 90 91 92 93 94 95 96 97 98 99 100